LSAT and Law School Admissions Forum

Get expert LSAT preparation and law school admissions advice from PowerScore Test Preparation.

User avatar
 Dave Killoran
PowerScore Staff
  • PowerScore Staff
  • Posts: 5852
  • Joined: Mar 25, 2011
|
#46080
Complete Question Explanation
(The complete setup for this game can be found here: lsat/viewtopic.php?t=10395)

The correct answer choice is (D)

Answer choices (A) and (B): As established in the game analysis, either M or P must be the sole member who serves on all three subcommittees. If either F or H served on all three, there would be a violation of one of the last two rules. Consequently, both of these answer choices are incorrect.

Answer choice (C): If G serves on every subcommittee that M and P serve on, and we know that one of M and P serves on all three subcommittees, then G must serve on all three subcommittees as well. This scenario violates the numerical distribution, and is therefore incorrect.

Answer choice (D): This is the correct answer.

Answer choice (E): This answer choice creates a scenario where M and P serve on exactly the same subcommittees. Since only one of M and P serves on all three subcommittees, and the other serves on fewer subcommittees, this answer choice is incorrect.

Get the most out of your LSAT Prep Plus subscription.

Analyze and track your performance with our Testing and Analytics Package.